Quasiconcavification


5

$ F_1, f_2 $ दो सुचारू कड़ाई-क्वासिकोक्वेव कार्य करें।   क्या वहाँ हमेशा मौजूद है मोनोटोन ट्रांसफॉर्मेशन $ g_1, g_2 $ जैसे कि योग   $ g_1 \ circ f_1 + g_2 \ circ f_2 $ एक कड़ाई से-क्वासिकोक्वेव फ़ंक्शन है?

जबकि यह प्रश्न गणितीय है, इसकी प्रेरणा आर्थिक है। कड़ाई से उत्तल वरीयताओं वाले दो लोगों को देखते हुए, हम एक सामाजिक कल्याण समारोह द्वारा उनकी सामान्य प्राथमिकताओं का प्रतिनिधित्व करना चाहते हैं। ऐसा करने का एक स्वाभाविक तरीका यह है कि प्रत्येक व्यक्ति की वरीयताओं को कुछ कड़ाई से-क्वैश्चनक्वेव फ़ंक्शन द्वारा दर्शाया जाए और उनकी राशि ली जाए। हालांकि, कड़ाई-क्वासिकोक्वेव कार्यों का योग अनिवार्य रूप से क्वासिकोस्केव नहीं है। तो यह दिलचस्प है कि क्या हम हमेशा विशिष्ट प्रतिनिधि कार्यों को पा सकते हैं जैसे कि योग है quasiconcave?

मुझे पहले से ही पता है कि, "सख्ती" के बिना, उत्तर नहीं है। यहाँ एक उदाहरण है: enter image description here

दोनों फ़ंक्शन कमजोर-क्वासिकोक्वेव हैं। जब एक फ़ंक्शन बढ़ रहा है, तो दूसरा फ्लैट है, और जब दूसरा घट रहा है, तो पहला फ्लैट है। इसलिए, हम उन पर क्या परिवर्तन लागू करते हैं, इस बात की परवाह किए बिना कि राशि एक लहर की तरह दिखेगी और क्वासाइन्सकेव नहीं होगी।

नोट: मैंने यह प्रश्न पूछा है MathOverflow पर कुछ समय पहले। Taneli Huuskonen कमज़ोर-क्वासिकोक्वेव फ़ंक्शन के लिए नकारात्मक उदाहरण के साथ आया था, लेकिन कड़ाई-क्वासिकोक्वेव फ़ंक्शन के लिए प्रश्न अभी भी खुला है। क्वासिकोक्वेव कार्यों के आर्थिक महत्व के कारण, मैंने सोचा कि यहां किसी के पास एक उत्तर हो सकता है।


यह हाल ही में एक पेपर के स्वाद में समान है, rasmusen.org/papers/quasi-connell-rasmusen.pdf मुझे संदेह है कि आपके प्रश्न का उत्तर आम तौर पर नहीं है

1
@AitakereCity वास्तव में, उनका पेपर पहली चीज़ थी जो मैंने कोशिश की थी। यदि $ f_1 $ और $ f_2 $ उनकी परिभाषा के अनुसार "अवशिष्ट" हैं, तो वे मेरी परिभाषा के अनुसार "क्वैसैन्सेवेबिल" हैं। $ G_1 $ और $ g_2 $ ऐसे चुनें कि $ g_i \ circ f_i $ अवतल हो। फिर, योग भी अवतल होता है, इसलिए योग अर्धचालकों का होता है। हालांकि, कुछ फ़ंक्शन कंफर्टेबल नहीं हैं, इसलिए यह एप्रोच काम नहीं करता है। मेरी हालत कमज़ोर है, इसलिए उम्मीद है कि यह हमेशा किया जा सकता है।
Erel Segal-Halevi

सही, हाँ यह एक दिलचस्प समस्या है। मैं इसके बारे में थोड़ा सोच रहा हूं। कमजोर मामले के लिए प्रतिरूप अच्छा था, और यह मुझे मेरे अनुमान / संदेह से ऊपर का प्रश्न बनाता है। मैंने ऊपरी समोच्च सेटों के संदर्भ में भी संपर्क करने की कोशिश की, और यह दृष्टिकोण अधिक फलदायी हो सकता है।

जवाबों:


2

एक आयाम में यदि कार्य करता है $ F_i $ बाध्य हैं, उदा। एक कॉम्पैक्ट अंतराल पर परिभाषित, फिर हाँ। लेना $ G_1 $ ऐसा है कि यह बनाता है $ F_1 $ लगभग स्थिर, उदा। $ g_1 (x) = \ frac {x} {N (\ max f_1- \ min f_1)} $ कहा पे $ एन & gt; 0 $ बड़ा है। लेना $ G_2 (x) = x $ फिर $ g_1 \ circ f_1 + g_2 \ circ f_2 \ लगभग f_2 $ , जो सख्ती से धारणा द्वारा quasiconcave है।

एक आयाम में एक क्वींसकेंव फंक्शन या तो सख्ती से बढ़ रहा है, सख्ती से कम हो रहा है या एकल-शिखर है।


दिलचस्प। यह केवल एक आयाम में क्यों काम करता है?
Erel Segal-Halevi

मैं यह दावा नहीं करता कि यह केवल एक आयाम में काम करता है, बस एक आयाम इसके काम करने के लिए पर्याप्त स्थिति है। संभवतः एक ही `सपाट तर्क 'का उपयोग किसी भी संख्या में आयामों में किया जा सकता है यदि फ़ंक्शन बाध्य हैं। निर्बाध कार्यों के साथ, समस्याएं हो सकती हैं।
Sander Heinsalu
हमारी साइट का प्रयोग करके, आप स्वीकार करते हैं कि आपने हमारी Cookie Policy और निजता नीति को पढ़ और समझा लिया है।
Licensed under cc by-sa 3.0 with attribution required.